Micromass' big integral challenge

In summary, this conversation discussed the interesting topic of integrals and different methods for solving them. A challenge was given for 10 integrals, with the rule that a correct solution must not only be given, but also a detailed explanation. The use of outside sources was allowed, but directly typing in the integral in wolframalpha was not permitted. The conversation included the solutions for some of the integrals, with different participants providing their own methods and explanations for solving them.
  • #1
micromass
Staff Emeritus
Science Advisor
Homework Helper
Insights Author
22,183
3,321
Integrals are pretty interesting, and there are a lot of different methods to solve them. In this thread, I will give as a challenge 10 integrals. Here are the rules:

  • For a solution to count, the answer must not only be correct, but a detailed solution must also be given.
  • A correct answer would consist of the correct number (which may be infinite) or the statement that the given integral does not exist.
  • Any use of outside sources is allowed, but do not look up the integral directly. For example, it is ok to go check calculus textbooks for methods, but it is not allowed to type in the integral in wolframalpha.
  • If you previously encountered this integral and remember the solution, then you cannot participate with that specific integral.
  • All mathematical methods are allowed.

Anyway, here we go:

  1. [itex]\int_0^1 \frac{\text{ln}(1+x)}{1+x^2}dx = \frac{\pi}{4}\text{ln}(\sqrt{2})[/itex] SOLVED BY Ssnow
  2. [itex]\int_0^\infty \frac{|\sin(x)|}{x}dx = +\infty[/itex] SOLVED BY PeroK
  3. [itex]\int_0^\infty \frac{\text{atan}(2016x) - \text{atan}(1916x)}{x}dx = \frac{\pi}{2}\text{ln} \left(\frac{2016}{1916}\right)[/itex] SOLVED BY Samy_A
  4. [itex]\int_0^{\pi/2} \frac{\sqrt{\sin(x)}}{\sqrt{\sin(x)}+\sqrt{\cos(x)}}dx = \frac{\pi}{4}[/itex] SOLVED BY Ssnow
  5. [itex]\int_0^1 \sqrt{-\text{ln}(x)}dx = \frac{\sqrt{\pi}}{2}[/itex] SOLVED BY Ssnow
  6. [itex]\int_0^1 \frac{1-4x^5}{(x^5 - x + 1)^2}dx = 1[/itex] SOLVED BY Samy_A
  7. [itex]\int_0^{\pi/2} \text{acos}\left(\frac{\cos(x)}{1+2\cos(x)}\right)dx[/itex]
  8. [itex]\int_0^\infty \frac{\sin^{9}(x)}{x}dx = \frac{35}{256}\pi[/itex] SOLVED BY vela
  9. [itex]\int_0^\infty \frac{x^{1916}}{x^{2016} + 1}dx = \frac{1}{2016} \pi \frac{1}{\sin{\frac{1917 \pi}{2016}}}[/itex] SOLVED BY fresh_42
  10. [itex]\int_{\sqrt{2}}^\infty \frac{1}{x + x^{\sqrt{2}}}dx = -\ln \frac{\sqrt{2}}{{(1+{\sqrt 2}^{\sqrt{2}-1}})^{\sqrt{2}+1}}[/itex] SOLVED BY Samy_A
 
Last edited:
  • Like
Likes Incand, StudentOfScience, Greg Bernhardt and 1 other person
Physics news on Phys.org
  • #2
I want propose the solution of 5. ##\int_{0}^{1}\sqrt{-\ln{x}}dx=\frac{\sqrt{\pi}}{2}##

I started changing ##-\ln{x}=t^{2}## so ##x=e^{-t^{2}}## and ##dx=-2te^{-t^{2}}dt##, changing also the domain:

##\int_{0}^{\infty}2t^{2}e^{-t^{2}}dt##

doing integration by parts ## t\rightarrow 1## and ## \int 2te^{-t^{2}}dt \rightarrow -e^{-t^{2}} ## so

##=-\lim_{z\rightarrow +\infty}te^{-t^{2}}|_{0}^{z}-\int_{0}^{\infty}-e^{-t^{2}}dt=0+\int_{0}^{\infty}e^{-t^{2}}dt=\frac{\sqrt{\pi}}{2}##

The last is half the Gaussian integral. I declare I never see this integral before ... It is correct?
 
  • Like
Likes ShayanJ
  • #3
Ssnow said:
I want propose the solution of 5. ##\int_{0}^{1}\sqrt{-\ln{x}}dx=\frac{\sqrt{\pi}}{2}##

I started changing ##-\ln{x}=t^{2}## so ##x=e^{-t^{2}}## and ##dx=-2te^{-t^{2}}dt##, changing also the domain:

##\int_{0}^{\infty}2t^{2}e^{-t^{2}}dt##

doing integration by parts ## t\rightarrow 1## and ## \int 2te^{-t^{2}}dt \rightarrow -e^{-t^{2}} ## so

##=-\lim_{z\rightarrow +\infty}te^{-t^{2}}|_{0}^{z}-\int_{0}^{\infty}-e^{-t^{2}}dt=0+\int_{0}^{\infty}e^{-t^{2}}dt=\frac{\sqrt{\pi}}{2}##

The last is half the Gaussian integral. I declare I never see this integral before ... It is correct?

Correct! Congratulations!
 
  • Like
Likes Ssnow
  • #4
The integral in #2 must diverge. For non-negative integer ##n##:

##\int_{n\pi}^{(n+1)\pi} \frac{|sin x|}{x} dx > \int_{n\pi}^{(n+1)\pi} \frac{|sin x|}{(n+1)\pi} dx = \frac{2}{(n+1)\pi}##

Hence:

##\int_{0}^{\infty} \frac{|sin x|}{x} dx > \sum_{n=0}^{\infty}\frac{2}{(n+1)\pi} = \frac{2}{\pi}\sum_{n=1}^{\infty} \frac{1}{n}##
 
  • Like
Likes Samy_A and ShayanJ
  • #5
PeroK said:
The integral in #2 must diverge. For non-negative integer ##n##:

##\int_{n\pi}^{(n+1)\pi} \frac{|sin x|}{x} dx > \int_{n\pi}^{(n+1)\pi} \frac{|sin x|}{(n+1)\pi} dx = \frac{2}{(n+1)\pi}##

Hence:

##\int_{0}^{\infty} \frac{|sin x|}{x} dx > \sum_{n=0}^{\infty}\frac{2}{(n+1)\pi} = \frac{2}{\pi}\sum_{n=1}^{\infty} \frac{1}{n}##

Correct!
 
  • #6
##\displaystyle \int_0^\infty \frac{\text{atan}(2016x) - \text{atan}(1916x)}{x}dx##

I use the general result ##\displaystyle \int_0^\infty \frac{f(ax)-f(bx)}{x} dx= (f(\infty) -f(0)) \log \frac{a}{b}## (proven using Leibnitz rule, see below).

So ##\displaystyle \int_0^\infty \frac{\text{atan}(2016x) - \text{atan}(1916x)}{x}dx =(\text{atan}( \infty ) - \text{atan}( 0)) \log \frac{2016}{1916} = \frac{\pi}{2} \log \frac{2016}{1916}##.

Now for ##\displaystyle \int_0^\infty \frac{f(ax)-f(bx)}{x} dx= (f(\infty) -f(0)) \log \frac{a}{b}## (*)
Set ##\displaystyle I(y)= \int_0^\infty \frac{f(yx)-f(bx)}{x} dx##

Notice that ##I(a)## is the integral we are trying to determine, and ##I(b)=0##.

Then ##\displaystyle I'(y) = \int_0^\infty f'(yx) dx##. Let's do the substitution ##z=yx##, so that ##\displaystyle I'(y)= \frac{1}{y}\int_0^\infty f'(z) dz =\frac{1}{y} (f(\infty) -f(0))##. Let's name ##A=f(\infty) -f(0)##.
We then have ##I'(y)=A \frac{1}{y}##, a differential equation with solution ##I(y)=A\log y + C##.
Now, ##0=I(b)= A\log b + C##, so that ##C=-A\log b##.
Finally, we have ##\displaystyle I(a)= A\log a + C = A\log a -A\log b= A \log \frac{a}{b}##, proving (*).

(Remark: in this proof, I made some implicit assumptions about ##f## and ##a,b##, assumptions clearly satisfied by the explicit integral we had to solve here.)
 
  • Like
Likes ShayanJ and PeroK
  • #7
Samy_A said:
##\displaystyle \int_0^\infty \frac{\text{atan}(2016x) - \text{atan}(1916x)}{x}dx##

I use the general result ##\displaystyle \int_0^\infty \frac{f(ax)-f(bx)}{x} dx= (f(\infty) -f(0)) \log \frac{a}{b}## (proven using Leibnitz rule, see below).

So ##\displaystyle \int_0^\infty \frac{\text{atan}(2016x) - \text{atan}(1916x)}{x}dx =(\text{atan}( \infty ) - \text{atan}( 0)) \log \frac{2016}{1916} = \frac{\pi}{2} \log \frac{2016}{1916}##.

Now for ##\displaystyle \int_0^\infty \frac{f(ax)-f(bx)}{x} dx= (f(\infty) -f(0)) \log \frac{a}{b}## (*)
Set ##\displaystyle I(y)= \int_0^\infty \frac{f(yx)-f(bx)}{x} dx##

Notice that ##I(a)## is the integral we are trying to determine, and ##I(b)=0##.

Then ##\displaystyle I'(y) = \int_0^\infty f'(yx) dx##. Let's do the substitution ##z=yx##, so that ##\displaystyle I'(y)= \frac{1}{y}\int_0^\infty f'(z) dz =\frac{1}{y} (f(\infty) -f(0))##. Let's name ##A=f(\infty) -f(0)##.
We then have ##I'(y)=A \frac{1}{y}##, a differential equation with solution ##I(y)=A\log y + C##.
Now, ##0=I(b)= A\log b + C##, so that ##C=-A\log b##.
Finally, we have ##\displaystyle I(a)= A\log a + C = A\log y -A\log b= A \frac{a}{b}##, proving (*).

Very nice!
 
  • #8
Looks fun. I only just started integration, but I'm going to bookmark this for a future challenge!
 
  • #9
I propose the solution for the 4. ## \int_{0}^{\frac{\pi}{2}}\frac{\sqrt{\sin{x}}}{\sqrt{\sin{x}}+\sqrt{\cos{x}}}dx=\frac{\pi}{4}##

The integral is ##\int_{0}^{\pi/2}\frac{1}{1+\sqrt{\cot{x}}}dx##, I start considering the substitution ##\cot{x}=t^{2}##, so the indefinite integral is:

## \int \frac{-2t}{(1+t)(1+t^{4})}dt##

where ##\cot{x}=t^{2}## and ##dx=-\frac{1}{1+t^{4}}2tdt##. I started with partial fraction:

##\frac{-2t}{(1+t)(1+t^{4})}= \frac{A}{1+t}+\frac{Bt^{3}+Ct^{2}+Dt+E}{1+t^{4}} ##

obtaining ##=\int \frac{1}{1+t}+\int \frac{-t^{3}+t^{2}-t-1}{1+t^{4}} ## that it is:

## \ln{(1+t)}-\frac{1}{4}\ln{(1+t^{4})} + \int \frac{t^{2}-t-1}{1+t^{4}}dt##

for the last integral I repeat the partial fraction:

##\frac{t^{2}-t-1}{1+t^{4}}=\frac{At+B}{t^{2}+t\sqrt{2}+1}+\frac{Ct+D}{t^{2}-t\sqrt{2}+1}##

and I find that ## A=-\frac{1}{\sqrt{2}},B=-(1+\sqrt{2})/2\sqrt{2}, C=1/\sqrt{2},D=(1-\sqrt{2})/2\sqrt{2}##, after I must adjust the two in order to have the derivative in the numerator, so obtaining the logaritm from both:

## -\frac{1}{2\sqrt{2}}\ln{(t^{2}+t\sqrt{2}+1)}## and from the second ##+\frac{1}{2\sqrt{2}}\ln{(t^{2}-t\sqrt{2}+1)}##,

what remain are two integrals ##\int \frac{-1/(2\sqrt{2})}{t^{2}+t\sqrt{2}+1}dt## and ##\int \frac{1/(2\sqrt{2})}{t^{2}-t\sqrt{2}+1}dt##, they gives two factors with sum ##-\arctan{(1+\sqrt{2}t)}##

so substituting ##t=\sqrt{\cot{x}}## and valutating between ##0## and ##\frac{\pi}{2}## (taking the limit where there are logaritm with ##\infty## because the ##\cot{0}=\infty##), I obtained the result ##\pi/2-\arctan{1}## that is ##\frac{\pi}{4}##. It is correct?
 
  • Like
Likes Samy_A and ShayanJ
  • #10
Ssnow said:
I propose the solution for the 4. ## \int_{0}^{\frac{\pi}{2}}\frac{\sqrt{\sin{x}}}{\sqrt{\sin{x}}+\sqrt{\cos{x}}}dx=\frac{\pi}{4}##

The integral is ##\int_{0}^{\pi/2}\frac{1}{1+\sqrt{\cot{x}}}dx##, I start considering the substitution ##\cot{x}=t^{2}##, so the indefinite integral is:

## \int \frac{-2t}{(1+t)(1+t^{4})}dt##

where ##\cot{x}=t^{2}## and ##dx=-\frac{1}{1+t^{4}}2tdt##. I started with partial fraction:

##\frac{-2t}{(1+t)(1+t^{4})}= \frac{A}{1+t}+\frac{Bt^{3}+Ct^{2}+Dt+E}{1+t^{4}} ##

obtaining ##=\int \frac{1}{1+t}+\int \frac{-t^{3}+t^{2}-t-1}{1+t^{4}} ## that it is:

## \ln{(1+t)}-\frac{1}{4}\ln{(1+t^{4})} + \int \frac{t^{2}-t-1}{1+t^{4}}dt##

for the last integral I repeat the partial fraction:

##\frac{t^{2}-t-1}{1+t^{4}}=\frac{At+B}{t^{2}+t\sqrt{2}+1}+\frac{Ct+D}{t^{2}-t\sqrt{2}+1}##

and I find that ## A=-\frac{1}{\sqrt{2}},B=-(1+\sqrt{2})/2\sqrt{2}, C=1/\sqrt{2},D=(1-\sqrt{2})/2\sqrt{2}##, after I must adjust the two in order to have the derivative in the numerator, so obtaining the logaritm from both:

## -\frac{1}{2\sqrt{2}}\ln{(t^{2}+t\sqrt{2}+1)}## and from the second ##+\frac{1}{2\sqrt{2}}\ln{(t^{2}-t\sqrt{2}+1)}##,

what remain are two integrals ##\int \frac{-1/(2\sqrt{2})}{t^{2}+t\sqrt{2}+1}dt## and ##\int \frac{1/(2\sqrt{2})}{t^{2}-t\sqrt{2}+1}dt##, they gives two factors with sum ##-\arctan{(1+\sqrt{2}t)}##

so substituting ##t=\sqrt{\cot{x}}## and valutating between ##0## and ##\frac{\pi}{2}## (taking the limit where there are logaritm with ##\infty## because the ##\cot{0}=\infty##), I obtained the result ##\pi/2-\arctan{1}## that is ##\frac{\pi}{4}##. It is correct?

That is the correct solution! Very well done!

An easier (but much trickier!) solution. Let ##I = \int_0^{\pi/2}\frac{\sqrt{\sin{x}}}{\sqrt{\sin{x}}+\sqrt{\cos{x}}}dx##. Perform substitution ##y = \pi/2 - x##, then
[tex]I = \int_0^{\pi/2} \frac{\sqrt{\cos{y}}}{\sqrt{\sin{y}}+\sqrt{\cos{y}}}dy[/tex]
Hence
[tex]2I = \int_0^{\pi/2} \frac{\sqrt{\sin{x}}+\sqrt{\cos{x}}}{\sqrt{\sin{x}}+\sqrt{\cos{x}}}dx = \int_0^{\pi/2} dx = \frac{\pi}{2}[/tex]
Thus, ##I = \frac{\pi}{4}##.
 
  • Like
Likes member 587159, ProfuselyQuarky, Samy_A and 2 others
  • #11
I propose the solution of 1. ##\int_{0}^{1}\frac{\ln{(1+x)}}{1+x^{2}}=\frac{\pi}{4}\ln{\sqrt{2}}##

I started substituting ##x=\tan{t}## so :

##\int_{0}^{\frac{\pi}{4}}\frac{\ln{(1+\tan{t})}}{1+\tan^{2}{t}}\frac{dt}{\cos^{2}{t}}##

that is by the fundamental relation:

##=\int_{0}^{\frac{\pi}{4}}\ln{(1+\tan{t})}dt##

Now ##1+\tan{t}=\frac{\cos{t}+\sin{t}}{\cos{t}}=\frac{sin{\left(\frac{\pi}{2}-t\right)}+\sin{t}}{\cos{t}}=\frac{2\sin{\pi/4}\cos{\left(t-\frac{\pi}{4}\right)}}{\cos{t}}=\frac{\sqrt{2}\cos{\left(t-\frac{\pi}{4}\right)}}{\cos{t}}##

so

##=\int_{0}^{\frac{\pi}{4}}\ln{\sqrt{2}}dt+\int_{0}^{\frac{\pi}{4}}\ln{\cos{(t-\pi/4)}}dt-\int_{0}^{\pi/4}\ln{\cos{t}}dt##

The second setting ##\frac{\pi}{4}-t=u## is

##=\int_{0}^{\frac{\pi}{4}}\ln{\cos{u}}du##

so it cancel with the last and the integral is ##=\int_{0}^{\frac{\pi}{4}}\ln{\sqrt{2}}dt=\frac{\pi}{4}\ln{\sqrt{2}}##
 
  • Like
Likes ShayanJ
  • #12
Ssnow said:
I propose the solution of 1. ##\int_{0}^{1}\frac{\ln{(1+x)}}{1+x^{2}}=\frac{\pi}{4}\ln{\sqrt{2}}##

I started substituting ##x=\tan{t}## so :

##\int_{0}^{\frac{\pi}{4}}\frac{\ln{(1+\tan{t})}}{1+\tan^{2}{t}}\frac{dt}{\cos^{2}{t}}##

that is by the fundamental relation:

##=\int_{0}^{\frac{\pi}{4}}\ln{(1+\tan{t})}dt##

Now ##1+\tan{t}=\frac{\cos{t}+\sin{t}}{\cos{t}}=\frac{sin{\left(\frac{\pi}{2}-t\right)}+\sin{t}}{\cos{t}}=\frac{2\sin{\pi/4}\cos{\left(t-\frac{\pi}{4}\right)}}{\cos{t}}=\frac{\sqrt{2}\cos{\left(t-\frac{\pi}{4}\right)}}{\cos{t}}##

so

##=\int_{0}^{\frac{\pi}{4}}\ln{\sqrt{2}}dt+\int_{0}^{\frac{\pi}{4}}\ln{\cos{(t-\pi/4)}}dt-\int_{0}^{\pi/4}\ln{\cos{t}}dt##

The second setting ##\frac{\pi}{4}-t=u## is

##=\int_{0}^{\frac{\pi}{4}}\ln{\cos{u}}du##

so it cancel with the last and the integral is ##=\int_{0}^{\frac{\pi}{4}}\ln{\sqrt{2}}dt=\frac{\pi}{4}\ln{\sqrt{2}}##

Correct again! Amazing that you came up with the substitution ##x = \tan{t}##. Very clever!
 
  • Like
Likes Ssnow
  • #13
micromass said:
Amazing that you came up with the substitution ## x= \tan{t} ##. Very clever!
So its not just me!

Great thread micromass. Actually its starting to become an art exhibition!
 
  • Like
Likes micromass
  • #14
Half of my challenge problems are already solved. I did not expect this to happen so soon! The people who found these solutions are truly integral masters!

Anyway, I wish to make a little adertisement now for the book containing these problems. I got these problems (except 6 which is from Apostol) from the beautiful book "Inside interesting integrals" from Paul J. Nahin. So if you want to learn tricky solutions or are up for a challenge. This is the book for you!

https://www.amazon.com/dp/1493912763/?tag=pfamazon01-20
 
  • Like
Likes ProfuselyQuarky and ShayanJ
  • #15
##\displaystyle \int_{\sqrt{2}}^\infty \frac{1}{x + x^{\sqrt{2}}}dx##

##f(x)=\frac{1}{x + x^{\sqrt{2}}}##
Doing a sort of partial fraction decomposition, ##\displaystyle f(x)=\frac{1}{x}- \frac{x^{\sqrt{2}-2}}{1+x^{\sqrt{2}-1}}##

##\displaystyle \int f(x) dx =\int \frac{1}{x} dx -\int \frac{x^{\sqrt{2}-2}}{1+x^{\sqrt{2}-1}}dx##
⇒ ##\displaystyle \int f(x) dx = \ln {x} - \frac{1}{\sqrt{2}-1} \int \frac{ d(x^{\sqrt{2}-1})}{1+x^{\sqrt{2}-1}} + C##
⇒ ##\displaystyle \int f(x) dx = \ln {x} - (\sqrt{2}+1)\ln(1+x^{\sqrt{2}-1}) + C##
⇒ ##\displaystyle \int f(x) dx =\ln \frac{x}{{(1+x^{\sqrt{2}-1}})^{\sqrt{2}+1}}+ C##

⇒ ##\displaystyle \int_{\sqrt{2}}^\infty f(x) dx = -\ln \frac{\sqrt{2}}{{(1+{\sqrt 2}^{\sqrt{2}-1}})^{\sqrt{2}+1}} \approx 1.50633 ##

I used that ##\displaystyle \lim_{x \rightarrow +\infty} \ln \frac{x}{{(1+x^{\sqrt{2}-1}})^{\sqrt{2}+1}}=0## so that the term at ##+ \infty## vanishes.
Proof for that limit:
set ##y=x^{\sqrt{2}-1}##
⇒ ##y^{\sqrt{2}+1}=x^{(\sqrt{2}-1)(\sqrt{2}+1)}=x##
⇒ ##\displaystyle \lim_{x \rightarrow +\infty} \frac{x}{{(1+x^{\sqrt{2}-1}})^{\sqrt{2}+1}}= \lim_{y \rightarrow +\infty} \frac{y^{\sqrt{2}+1}}{(1+y)^{\sqrt{2}+1}} = \lim_{y \rightarrow +\infty} (\frac{y}{y+1})^{\sqrt{2}+1}=1##
and hence the limit of the log of that nice expression will be 0.
 
Last edited:
  • Like
Likes Dougias, Ssnow and ShayanJ
  • #16
Samy_A said:
##\displaystyle \int_{\sqrt{2}}^\infty \frac{1}{x + x^{\sqrt{2}}}dx##

##f(x)=\frac{1}{x + x^{\sqrt{2}}}##
Doing a sort of partial fraction decomposition, ##\displaystyle f(x)=\frac{1}{x}- \frac{x^{\sqrt{2}-2}}{1+x^{\sqrt{2}-1}}##

##\displaystyle \int f(x) dx =\int \frac{1}{x} dx -\int \frac{x^{\sqrt{2}-2}}{1+x^{\sqrt{2}-1}}dx##
⇒ ##\displaystyle \int f(x) dx = \ln {x} - \frac{1}{\sqrt{2}-1} \int \frac{ d(x^{\sqrt{2}-1})}{1+x^{\sqrt{2}-1}} + C##
⇒ ##\displaystyle \int f(x) dx = \ln {x} - (\sqrt{2}+1)\ln(1+x^{\sqrt{2}-1}) + C##
⇒ ##\displaystyle \int f(x) dx =\ln \frac{x}{{(1+x^{\sqrt{2}-1}})^{\sqrt{2}+1}}+ C##

⇒ ##\displaystyle \int_{\sqrt{2}}^\infty f(x) dx = -\ln \frac{\sqrt{2}}{{(1+{\sqrt 2}^{\sqrt{2}-1}})^{\sqrt{2}+1}} \approx 1.50633 ##

I used that ##\displaystyle \lim_{x \rightarrow +\infty} \ln \frac{x}{{(1+x^{\sqrt{2}-1}})^{\sqrt{2}+1}}=0## so that the term at ##+ \infty## vanishes.
Proof for that limit:
set ##y=x^{\sqrt{2}-1}##
⇒ ##y^{\sqrt{2}+1}=x^{(\sqrt{2}-1)(\sqrt{2}+1)}=x##
⇒ ##\displaystyle \lim_{x \rightarrow +\infty} \frac{x}{{(1+x^{\sqrt{2}-1}})^{\sqrt{2}+1}}= \lim_{y \rightarrow +\infty} \frac{y^{\sqrt{2}+1}}{(1+y)^{\sqrt{2}+1}} = \lim_{y \rightarrow +\infty} (\frac{y}{y+1})^{\sqrt{2}+1}=1##
and hence the limit of the log of that nice expression will be 0.

Very nice solution!
 
  • #17
\begin{align*}
\int_0^\infty \frac{\sin^9 x}{x}\,dx &= \frac 12 \int_{-\infty}^\infty \frac{\sin^9 x}{x}\,dx \\
&= \frac 12 \frac{1}{(2i)^8} \int_{-\infty}^\infty \frac{1}{2i}\frac{(e^{ix}-e^{-ix})^9}{x}\,dx \\
&= \frac {1}{512} \int_{-\infty}^\infty \frac{1}{2i}\left(\frac{e^{i9x}-9 e^{i7x} + 36 e^{i5x} - 84 e^{i3x} + 126 e^{ix} - 126 e^{-ix} + 84e^{-i3x} -36 e^{-i5x} + 9 e^{-i7x} - e^{-i9x}}{x}\right)\,dx \\
&= \frac {1}{512} \int_{-\infty}^\infty \left(\frac{\sin 9x}{x} - 9 \frac{\sin 7x}{x} + 36 \frac{\sin 5x}{x} - 84 \frac{\sin 3x}{x} + 126\frac{\sin x}{x}\right)\,dx \\
&= \frac{1}{512} (1-9+36-84+126)\int_{-\infty}^\infty \frac{\sin u}{u}\,du \\
&= \frac{35}{256}\pi
\end{align*}
 
  • Like
Likes Ssnow, Samy_A and ShayanJ
  • #18
vela said:
\begin{align*}
\int_0^\infty \frac{\sin^9 x}{x}\,dx &= \frac 12 \int_{-\infty}^\infty \frac{\sin^9 x}{x}\,dx \\
&= \frac 12 \frac{1}{(2i)^8} \int_{-\infty}^\infty \frac{1}{2i}\frac{(e^{ix}-e^{-ix})^9}{x}\,dx \\
&= \frac {1}{512} \int_{-\infty}^\infty \frac{1}{2i}\left(\frac{e^{i9x}-9 e^{i7x} + 36 e^{i5x} - 84 e^{i3x} + 126 e^{ix} - 126 e^{-ix} + 84e^{-i3x} -36 e^{-i5x} + 9 e^{-i7x} - e^{-i9x}}{x}\right)\,dx \\
&= \frac {1}{512} \int_{-\infty}^\infty \left(\frac{\sin 9x}{x} - 9 \frac{\sin 7x}{x} + 36 \frac{\sin 5x}{x} - 84 \frac{\sin 3x}{x} + 126\frac{\sin x}{x}\right)\,dx \\
&= \frac{1}{512} (1-9+36-84+126)\int_{-\infty}^\infty \frac{\sin u}{u}\,du \\
&= \frac{35}{256}\pi
\end{align*}

Very cool solution! Shows how complex numbers make life a lot easier!
 
  • #19
micromass said:
Very cool solution! Shows how complex numbers make life a lot easier!
Does that mean you had a more complicated method in mind?
Although vela's method is indeed cool, I'd like to see other methods too.
 
  • #20
Shyan said:
Does that mean you had a more complicated method in mind?
Although vela's method is indeed cool, I'd like to see other methods too.

No, this is the method I had in mind. If you wish to find a solution without complex numbers, then you'll need to find another way to prove
[tex]\sin^9x = \sin(9x) - 9\sin(7x) + 36\sin(5x) - 84\sin(3x) + 126\sin(x)[/tex]
It is not very difficult to imagine how to prove or find this formula without complex numbers, you just use the addition formula on ##\sin(nx)## a few times. But using the complex exponentials seems to be a lot more elegant!
 
  • Like
Likes ShayanJ
  • #21
vela said:
\begin{align*}
\int_0^\infty \frac{\sin^9 x}{x}\,dx &= \frac 12 \int_{-\infty}^\infty \frac{\sin^9 x}{x}\,dx \\
&= \frac 12 \frac{1}{(2i)^8} \int_{-\infty}^\infty \frac{1}{2i}\frac{(e^{ix}-e^{-ix})^9}{x}\,dx \\
&= \frac {1}{512} \int_{-\infty}^\infty \frac{1}{2i}\left(\frac{e^{i9x}-9 e^{i7x} + 36 e^{i5x} - 84 e^{i3x} + 126 e^{ix} - 126 e^{-ix} + 84e^{-i3x} -36 e^{-i5x} + 9 e^{-i7x} - e^{-i9x}}{x}\right)\,dx \\
&= \frac {1}{512} \int_{-\infty}^\infty \left(\frac{\sin 9x}{x} - 9 \frac{\sin 7x}{x} + 36 \frac{\sin 5x}{x} - 84 \frac{\sin 3x}{x} + 126\frac{\sin x}{x}\right)\,dx \\
&= \frac{1}{512} (1-9+36-84+126)\int_{-\infty}^\infty \frac{\sin u}{u}\,du \\
&= \frac{35}{256}\pi
\end{align*}
Very cool indeed.

I gave this one a look, and my "never failing intuition" told me that it will probably be solvable with contour integration. It may well be, but this is much nicer.
 
  • #22
Samy_A said:
I gave this one a look, and my "never failing intuition" told me that it will probably be solvable with contour integration. It may well be, but this is much nicer.
Contour integration was my first thought too, which is why my first step was to change the limits of the integral. I guess I did assume that the answer to the last integral is known, but that could be evaluated using contour integration.
 
  • Like
Likes Ssnow
  • #23
Samy_A said:
I gave this one a look, and my "never failing intuition" told me that it will probably be solvable with contour integration. It may well be, but this is much nicer.

I thought some special function as ##Si(z)=\int_{0}^{z}\frac{\sin{t}}{t}dt## but yes always pass by the complex street ...
 
  • #24
Thinking your trick about previous integral I want propose the solution of 7.

Let ## I=\int_{0}^{\frac{\pi}{2}}\arccos{\left(\frac{\cos{x}}{1+2\cos{x}}\right)}dx##

We observe that the function ##\frac{\cos{x}}{1+2\cos{x}}## is even so ##2I=\int_{-\frac{\pi}{2}}^{\frac{\pi}{2}}\arccos{\left(\frac{\cos{x}}{1+2\cos{x}}\right)}dx##

Now changing ##x\rightarrow -y## we have

##2I=\int_{\frac{\pi}{2}}^{-\frac{\pi}{2}}-\arccos{\frac{\cos{y}}{1+2\cos{y}}}dy##

and by the identity ##\arccos{(-x)}=\pi-\arccos{(x)}## we have that:

##2I=\int_{\frac{\pi}{2}}^{-\frac{\pi}{2}}\arccos{\frac{\cos{y}}{1+2\cos{y}}}-\pi dy=-2I+\int_{-\frac{\pi}{2}}^{\frac{\pi}{2}}\pi dy##

so ##4I=\frac{\pi^{2}}{2}+\frac{\pi^{2}}{2}## then ##I=\frac{\pi^{2}}{4}##.
 
  • Like
Likes ShayanJ
  • #25
Ssnow said:
Thinking your trick about previous integral I want propose the solution of 7.

Let ## I=\int_{0}^{\frac{\pi}{2}}\arccos{\left(\frac{\cos{x}}{1+2\cos{x}}\right)}dx##

We observe that the function ##\frac{\cos{x}}{1+2\cos{x}}## is even so ##2I=\int_{-\frac{\pi}{2}}^{\frac{\pi}{2}}\arccos{\left(\frac{\cos{x}}{1+2\cos{x}}\right)}dx##

Now changing ##x\rightarrow -y## we have

##2I=\int_{\frac{\pi}{2}}^{-\frac{\pi}{2}}-\arccos{\frac{\cos{y}}{1+2\cos{y}}}dy##

and by the identity ##\arccos{(-x)}=\pi-\arccos{(x)}## we have that:

##2I=\int_{\frac{\pi}{2}}^{-\frac{\pi}{2}}\arccos{\frac{\cos{y}}{1+2\cos{y}}}-\pi dy=-2I+\int_{-\frac{\pi}{2}}^{\frac{\pi}{2}}\pi dy##

so ##4I=\frac{\pi^{2}}{2}+\frac{\pi^{2}}{2}## then ##I=\frac{\pi^{2}}{4}##.

When you use ## \arccos(-x)=\pi-\arccos(x) ##, shouldn't you get ## \int_{\frac{\pi}{2}}^{-\frac{\pi}{2}} (\arccos{(\frac{-\cos{y}}{1+2\cos{y}}})-\pi ) dy ##? What did you do with that minus sign?
 
Last edited:
  • #26
Ssnow said:
Thinking your trick about previous integral I want propose the solution of 7.

Let ## I=\int_{0}^{\frac{\pi}{2}}\arccos{\left(\frac{\cos{x}}{1+2\cos{x}}\right)}dx##

We observe that the function ##\frac{\cos{x}}{1+2\cos{x}}## is even so ##2I=\int_{-\frac{\pi}{2}}^{\frac{\pi}{2}}\arccos{\left(\frac{\cos{x}}{1+2\cos{x}}\right)}dx##

Now changing ##x\rightarrow -y## we have

##2I=\int_{\frac{\pi}{2}}^{-\frac{\pi}{2}}-\arccos{\frac{\cos{y}}{1+2\cos{y}}}dy##

and by the identity ##\arccos{(-x)}=\pi-\arccos{(x)}## we have that:

##2I=\int_{\frac{\pi}{2}}^{-\frac{\pi}{2}}\arccos{\frac{\cos{y}}{1+2\cos{y}}}-\pi dy=-2I+\int_{-\frac{\pi}{2}}^{\frac{\pi}{2}}\pi dy##

so ##4I=\frac{\pi^{2}}{2}+\frac{\pi^{2}}{2}## then ##I=\frac{\pi^{2}}{4}##.

Sorry, this is not the correct answer. Number 7 is definitely the grand prize of this thread, it is very difficult!
 
  • #27
##\int_0^1 \frac{1-4x^5}{(x^5 - x + 1)^2}dx##

The derivative of quotient of polynomials, ##\frac{P}{Q}##, is ##\frac{QP'-PQ'}{Q²}##. The degree of the numerator being (in general) deg(P)+deg(Q)-1.
In our case, deg(Q)=5, so deg(P) should be 1, as deg(P)+deg(Q)-1=5.
That leads to an ansatz:
##\frac{1-4x^5}{(x^5 - x + 1)^2} =\frac{d}{dx} \frac{Ax+B}{x^5 - x + 1}##

Now, ##\frac{d}{dx} \frac{Ax+B}{x^5 - x + 1}= \frac{(x^5 - x + 1)A-(Ax+b)(5x^4-1)}{(x^5 - x + 1)^2}=\frac{Ax^5-Ax+A-5Ax^5+Ax-5Bx^4+B}{(x^5 - x + 1)^2}##
As we have no term in ##x^4## in the numerator, ##B=0##.
Hence ##\frac{d}{dx} \frac{Ax+B}{x^5 - x + 1}=\frac{Ax^5-Ax+A-5Ax^5+Ax}{(x^5 - x + 1)^2}=\frac{Ax^5+A-5Ax^5}{(x^5 - x + 1)^2}##
And now a miracle occurs, A must be one, and sure enough, the numerator becomes ##1-4x^5##.

So, ##\displaystyle \int_0^1 \frac{1-4x^5}{(x^5 - x + 1)^2}dx = \left. \frac{x}{x^5 - x + 1} \right|_0^1 =1##

(Latex fixed ...)
 
  • Like
Likes Ssnow and ShayanJ
  • #28
Samy_A said:
##\int_0^1 \frac{1-4x^5}{(x^5 - x + 1)^2}dx##

The derivative of quotient of polynomials, ##\frac{P}{Q}##, is ##\frac{QP'-PQ'}{Q²}##. The degree of the numerator being (in general) deg(P)+deg(Q)-1.
In our case, deg(Q)=5, so deg(P) should be 1, as deg(P)+deg(Q)-1=5.
That leads to an ansatz:
##\frac{1-4x^5}{(x^5 - x + 1)^2} =\frac{d}{dx} \frac{Ax+B}{x^5 - x + 1}##

Now, ##\frac{d}{dx} \frac{Ax+B}{x^5 - x + 1}= \frac{(x^5 - x + 1)A-(Ax+b)(5x^4-1)}{(x^5 - x + 1)^2}=\frac{Ax^5-Ax+A-5Ax^5+Ax-5Bx^4+B}{(x^5 - x + 1)^2}##
As we have no term in ##x^4## in the numerator, ##B=0##.
Hence ##\frac{d}{dx} \frac{Ax+B}{x^5 - x + 1}=\frac{Ax^5-Ax+A-5Ax^5+Ax}{(x^5 - x + 1)^2}=\frac{Ax^5+A-5Ax^5}{(x^5 - x + 1)^2}##
And now a miracle occurs, A must be one, and sure enough, the numerator becomes ##1-4x^5##.

So, ##\displaystyle \int_0^1 \frac{1-4x^5}{(x^5 - x + 1)^2}dx = \left. \frac{x}{x^5 - x + 1} \right|_0^1 =1##

(Latex fixed ...)

Perfect!
 
  • #29
Shyan said:
When you use arccos(−x)=π−arccos(x) \arccos(-x)=\pi-\arccos(x) , shouldn't you get ∫−π2π2(arccos(−cosy1+2cosy)−π)dy \int_{\frac{\pi}{2}}^{-\frac{\pi}{2}} (\arccos{(\frac{-\cos{y}}{1+2\cos{y}}})-\pi ) dy ? What did you do with that minus sign?

yes, thank you now I understand the error. I think the strategy is another ...
 
  • #30
#9: [itex]\int_0^\infty \frac{x^{1916}}{x^{2016} + 1}dx[/itex]

(I'm not sure if it is a valid solution since I looked up the Γ-function and its formulas ... after despaired on 2016th roots of unity and many ugly looking sheets of paper ...)

For convenience I take ##n=2016## and ##m=1916##.
The disturbing denominator can be transformed into an integral:

$$\int_0^\infty e^{(-x^n -1) y} dy = [- \frac{1}{x^n + 1} * e^{(-x^n -1) y } ]_{ y=0 }^{ y=\infty } = \frac{1}{x^n + 1}$$
Thus
$$\int_0^\infty \frac{x^{1916}}{x^{2016} + 1}dx = \int_0^\infty x^{m} \int_0^\infty e^{-x^n y} e^{-y} dy dx$$ and with Fubini's theorem
$$\int_0^\infty \frac{x^{1916}}{x^{2016} + 1}dx = \int_0^\infty\int_0^\infty x^m e^{-x^n y} e^{-y} dx dy$$.

Substitution ##z = x^n y## with ##dx = \frac{1}{ny} x^{1-n} dz## yields
$$\int_0^\infty \frac{x^{1916}}{x^{2016} + 1}dx = \frac{1}{n} \int_0^\infty e^{-y} (\frac{1}{y})^\frac{m+1}{n} dy \int_0^\infty e^{-z} z^\frac{m-n+1}{n} dz = \frac{1}{n} Γ(\frac{n-m-1}{n}) Γ(\frac{m+1}{n}) $$
$$\int_0^\infty \frac{x^{1916}}{x^{2016} + 1}dx = \frac{1}{n} Γ(1-\frac{m+1}{n}) Γ(\frac{m+1}{n}) = \frac{1}{2016} π \frac{1}{\sin{\frac{1917 π}{2016}}} ≈ 0.0101411901 $$
 
  • Like
Likes ShayanJ, Ssnow and Samy_A
  • #31
fresh_42 said:
#9: [itex]\int_0^\infty \frac{x^{1916}}{x^{2016} + 1}dx[/itex]

(I'm not sure if it is a valid solution since I looked up the Γ-function and its formulas ... after despaired on 2016th roots of unity and many ugly looking sheets of paper ...)

For convenience I take ##n=2016## and ##m=1916##.
The disturbing denominator can be transformed into an integral:

$$\int_0^\infty e^{(-x^n -1) y} dy = [- \frac{1}{x^n + 1} * e^{(-x^n -1) y } ]_{ y=0 }^{ y=\infty } = \frac{1}{x^n + 1}$$
Thus
$$\int_0^\infty \frac{x^{1916}}{x^{2016} + 1}dx = \int_0^\infty x^{m} \int_0^\infty e^{-x^n y} e^{-y} dy dx$$ and with Fubini's theorem
$$\int_0^\infty \frac{x^{1916}}{x^{2016} + 1}dx = \int_0^\infty\int_0^\infty x^m e^{-x^n y} e^{-y} dx dy$$.

Substitution ##z = x^n y## with ##dx = \frac{1}{ny} x^{1-n} dz## yields
$$\int_0^\infty \frac{x^{1916}}{x^{2016} + 1}dx = \frac{1}{n} \int_0^\infty e^{-y} (\frac{1}{y})^\frac{m+1}{n} dy \int_0^\infty e^{-z} z^\frac{m-n+1}{n} dz = \frac{1}{n} Γ(\frac{n-m-1}{n}) Γ(\frac{m+1}{n}) = \frac{1}{n} Γ(1-\frac{m+1}{n}) Γ(\frac{m+1}{n}) $$
$$\int_0^\infty \frac{x^{1916}}{x^{2016} + 1}dx = \frac{1}{2016} π \frac{1}{\sin{\frac{1917 π}{2016}}} ≈ 0.0101411901 $$

That is an extremely elegant solution! Congratulations! Don't worry about having looked up the ##\Gamma##-function and similar formulas, you were allowed to do that.
 
  • #32
This is a question regarding 7. This is not the answer.
I tried evaluating # 7 by finding an antiderivative and then using the Fundamental Theorem. I thought I found the antiderivative until I checked it on Desmos (I don't know if that is even allowed). I probably made a mistake that messed the whole problem up. Thus, I cannot be sure if the primitive can be represented in closed form with elementary functions. This is a lot to ask, but if anyone is willing to follow my work to see where I possibly went wrong, that would be great. The outline of my solution is as follows.
  1. Use weierstrass substitution
  2. Simplify
  3. Use integration by parts
    1. ## u= arccos(\frac{1-s^2}{3-s^2}) ##
    2. ## du=\frac{4s}{(3-s^2)^2*(1-(\frac{1-s^2}{3-s^2})^2)^\frac{1}{2}} ## (the bolded 2 is an exponent)
    3. ## dv=1+s^2 ##
  4. Simplify integrand resulting from the vdu section of integration by parts
  5. Substitute: ## p=3-s^2 ##
  6. Rewrite integrand in terms of p
  7. Simplify integrand
  8. Trig Substitution: ## p+1/2=\frac{\sec(q)}{2} ##
  9. Rewrite in terms of q
  10. Simplify
  11. Split numerator and integrate the remaining three integrands
    1. Note that all three follow the same pattern in how they were integrated; use long division and then integrate
  12. Back-substitute (I didn't do this since this would take a very long time and would be considerably tedious)
I did omit details in some parts (like in steps 2, 6, 9, and 11). If you want to see my full work, the link below should take you to a OneNote notebook with the (attempted) solution. If you would like me to put the details for 2, 6, 9, and 11 in, just let me know. Finally, kudos to all of you; you are way above me in integration (I'm only at the calc 2 level). I'm sure you could come up with an elegant solution that is less than a fourth of my work. Thank you for taking the time to read my post.
Notebook link: https://onedrive.live.com/redir?page=view&resid=B4AB9991BD097345!2597&authkey=!AFXWqJAgfdt0erA
Desmos link: https://www.desmos.com/calculator/vnvhapmyy1
 
Last edited:
  • #33
I'll actually give #7 a shot (even though I "tried" already). Also, I hope it's valid to check your answer with a graphing calculator. I'm probably wrong anyways, so don't regard my answer as much. This is what I have.

I think ##\int\arccos(\frac{cosx}{1+2cosx}) = 2[\arctan(s)]*[\arccos(\frac{1-s^2}{3-s^2})+\arcsin(\frac{1-s^2}{3-s^2})] + C ##, where ## s= \frac{sinx}{1+cosx}=tan(x/2) ##. I checked the antiderivative on the graphing calculator, but it was wrong since the derivative of the primitive didn't coincide with the integrand (see link: https://www.desmos.com/calculator/dpizsq7x7a). I will include work of how I obtained the primitive below. Before I do so, when I changed the limits of integration after the substitution, I got 0 and 1 as the limits. Using the antiderivative to evaluate the original definite integral yielded a value of ## \pi^2/4 ## This is probably wrong, but at least I tried.

How I found the (wrong) primitive:
1) Use the weierstrass substitution to rewrite the integrand as ##2 \int\arccos(\frac{1-s^2}{3-s^2})*\frac{1}{1+s^2}ds ##
2) Using integration by parts where ## u=\arccos(\frac{1-s^2}{3-s^2})## let's us write ##2 \int\arccos(\frac{1-s^2}{3-s^2})*\frac{1}{1+s^2}ds= 2[\arctan(s)][arccos(\frac{1-s^2}{3-s^2})]-2\int\frac{4s\arctan(s)}{(3-s^2)^2*(1-(\frac{1-s^2}{3-s^2})^2)^\frac{1}{2}}ds ##
3)To evaluate the remaining integral, use parts again with ##u=\arctan(s) ##. This allows us to say ##-2\int\frac{4s\arctan(s)}{(3-s^2)^2*(1-(\frac{1-s^2}{3-s^2})^2)^\frac{1}{2}}ds = 2(\arctan(s))*(arcsin(\frac{1-s^2}{3-s^2}))+2\int\frac{arcsin(\frac{1-s^2}{3-s^2}){1+s^2}ds ## For some reason, the LaTeX above is not working, so I will attach an image below:
Screen Shot 2016-04-23 at 2.29.50 PM.png
4) Now, let us solve the integral ##2\int\frac{\arcsin(\frac{1-s^2}{3-s^2})}{1+s^2}ds ##. Using parts once more where ##u=\arcsin(\frac{1-s^2}{3-s^2}) ## gives an interesting result. That is, we can combine like integrals (the procedure used to solve the integral ## \int\sec^3(x) ##). So, combining like integrals, we have that ##-2*\int\frac{4s\arctan(s)}{(3-s^2)^2*(1-(\frac{1-s^2}{3-s^2})^2)^\frac{1}{2}}ds =2[\arctan(s)}*[\arcsin(\frac{1-s^2}{3-s^2}) ## Again, the LaTeX may not be working, so here is another image (my apologies for attaching many images): Screen Shot 2016-04-23 at 2.44.36 PM.png
5) Thus, combining steps 2 and 3-4, we get the primitive ## 2[\arctan(s)]*[\arccos(\frac{1-s^2}{3-s^2})+\arcsin(\frac{1-s^2}{3-s^2})] + C ##

Thank you all for taking the time to read my work and putting up with me. Anyhow, I'm sure my work is wrong (and I've had 2 tries in addition to using a graphing calculator).
 

Attachments

  • Screen Shot 2016-04-23 at 2.29.50 PM.png
    Screen Shot 2016-04-23 at 2.29.50 PM.png
    11.2 KB · Views: 701
  • Screen Shot 2016-04-23 at 2.44.36 PM.png
    Screen Shot 2016-04-23 at 2.44.36 PM.png
    5.1 KB · Views: 745
  • #34
By the way, you can ignore post 32 since I found my error. I have not found my error in post 33 though.
 
  • #35
StudentOfScience said:
By the way, you can ignore post 32 since I found my error. I have not found my error in post 33 though.
You're at least close to the correct solution and as far as I can see with some correct ideas. However, it's hard for me to read your text and find eventual errors. The many substitutions needed in a brute force trial IMO require a careful and extended presentation (through formulas and equations rather than text). I still hope to find a "nice" trick instead of ever changing the variable and often return to the original integrand. Plus the substitution I've found to solve a similar integral isn't really nice and easy.
 

Similar threads

  • Math Proof Training and Practice
2
Replies
69
Views
4K
  • Topology and Analysis
Replies
4
Views
282
  • Math Proof Training and Practice
2
Replies
64
Views
12K
  • Math Proof Training and Practice
4
Replies
114
Views
6K
  • Math Proof Training and Practice
Replies
25
Views
2K
  • Math Proof Training and Practice
3
Replies
93
Views
10K
  • Math Proof Training and Practice
2
Replies
60
Views
8K
Replies
2
Views
301
  • General Math
Replies
2
Views
1K
Replies
1
Views
946
Back
Top